X has a Bernoulli distribution with parameter $p$ and Y has a Poisson distribution with mean $p$. What is the...












-1












$begingroup$


X ~ Bernoulli($p$) and Y ~ Poisson($p$) where $0<p<1$ and $p$ is the fixed number.



What is the maximum value of $P(X=Y)$ where the maximum value is taken over all possible joint distributions with specified marginal distributions?



That is... what is the joint distribution that maximizes the probability of X being equal to Y?



If I assume that $X$ and $Y$ are independent, finding the probability is easy since the only possible value $X$ can be is either $0$ or $1$. However, if I don't assume their independence, things get complicated. Any hints or help please?










share|cite|improve this question











$endgroup$












  • $begingroup$
    Since $P(X=0)=1-p<e^{-p}=P(Y=0)$, $P(X=1)=p>pe^{-p}=P(Y=1)$, and $P(Xgeqslant2)=0$, the maximum is $$1-p+pe^{-p}$$
    $endgroup$
    – Did
    Jan 13 at 20:30












  • $begingroup$
    @Did That comment should be an answer, I think
    $endgroup$
    – leonbloy
    Jan 13 at 20:31










  • $begingroup$
    @Did Are we adding the lower probability for each case to ensure the equality? like.. adding the upper bound for each case?
    $endgroup$
    – JHK
    Jan 13 at 20:45










  • $begingroup$
    Use the remark that $$P(X=Y)=sum_nP(X=n,Y=n)leqslantsum_nmin{P(X=n),P(Y=n)}$$ hence in the present case, $$P(X=Y)leqslant P(X=0)+P(Y=1)$$ and to complete the proof, it remains to build $(X,Y)$ such that the inequality is an equality.
    $endgroup$
    – Did
    Jan 13 at 20:46


















-1












$begingroup$


X ~ Bernoulli($p$) and Y ~ Poisson($p$) where $0<p<1$ and $p$ is the fixed number.



What is the maximum value of $P(X=Y)$ where the maximum value is taken over all possible joint distributions with specified marginal distributions?



That is... what is the joint distribution that maximizes the probability of X being equal to Y?



If I assume that $X$ and $Y$ are independent, finding the probability is easy since the only possible value $X$ can be is either $0$ or $1$. However, if I don't assume their independence, things get complicated. Any hints or help please?










share|cite|improve this question











$endgroup$












  • $begingroup$
    Since $P(X=0)=1-p<e^{-p}=P(Y=0)$, $P(X=1)=p>pe^{-p}=P(Y=1)$, and $P(Xgeqslant2)=0$, the maximum is $$1-p+pe^{-p}$$
    $endgroup$
    – Did
    Jan 13 at 20:30












  • $begingroup$
    @Did That comment should be an answer, I think
    $endgroup$
    – leonbloy
    Jan 13 at 20:31










  • $begingroup$
    @Did Are we adding the lower probability for each case to ensure the equality? like.. adding the upper bound for each case?
    $endgroup$
    – JHK
    Jan 13 at 20:45










  • $begingroup$
    Use the remark that $$P(X=Y)=sum_nP(X=n,Y=n)leqslantsum_nmin{P(X=n),P(Y=n)}$$ hence in the present case, $$P(X=Y)leqslant P(X=0)+P(Y=1)$$ and to complete the proof, it remains to build $(X,Y)$ such that the inequality is an equality.
    $endgroup$
    – Did
    Jan 13 at 20:46
















-1












-1








-1





$begingroup$


X ~ Bernoulli($p$) and Y ~ Poisson($p$) where $0<p<1$ and $p$ is the fixed number.



What is the maximum value of $P(X=Y)$ where the maximum value is taken over all possible joint distributions with specified marginal distributions?



That is... what is the joint distribution that maximizes the probability of X being equal to Y?



If I assume that $X$ and $Y$ are independent, finding the probability is easy since the only possible value $X$ can be is either $0$ or $1$. However, if I don't assume their independence, things get complicated. Any hints or help please?










share|cite|improve this question











$endgroup$




X ~ Bernoulli($p$) and Y ~ Poisson($p$) where $0<p<1$ and $p$ is the fixed number.



What is the maximum value of $P(X=Y)$ where the maximum value is taken over all possible joint distributions with specified marginal distributions?



That is... what is the joint distribution that maximizes the probability of X being equal to Y?



If I assume that $X$ and $Y$ are independent, finding the probability is easy since the only possible value $X$ can be is either $0$ or $1$. However, if I don't assume their independence, things get complicated. Any hints or help please?







probability probability-theory probability-distributions






share|cite|improve this question















share|cite|improve this question













share|cite|improve this question




share|cite|improve this question








edited Jan 13 at 20:43









Bernard

119k740113




119k740113










asked Jan 13 at 20:24









JHKJHK

12




12












  • $begingroup$
    Since $P(X=0)=1-p<e^{-p}=P(Y=0)$, $P(X=1)=p>pe^{-p}=P(Y=1)$, and $P(Xgeqslant2)=0$, the maximum is $$1-p+pe^{-p}$$
    $endgroup$
    – Did
    Jan 13 at 20:30












  • $begingroup$
    @Did That comment should be an answer, I think
    $endgroup$
    – leonbloy
    Jan 13 at 20:31










  • $begingroup$
    @Did Are we adding the lower probability for each case to ensure the equality? like.. adding the upper bound for each case?
    $endgroup$
    – JHK
    Jan 13 at 20:45










  • $begingroup$
    Use the remark that $$P(X=Y)=sum_nP(X=n,Y=n)leqslantsum_nmin{P(X=n),P(Y=n)}$$ hence in the present case, $$P(X=Y)leqslant P(X=0)+P(Y=1)$$ and to complete the proof, it remains to build $(X,Y)$ such that the inequality is an equality.
    $endgroup$
    – Did
    Jan 13 at 20:46




















  • $begingroup$
    Since $P(X=0)=1-p<e^{-p}=P(Y=0)$, $P(X=1)=p>pe^{-p}=P(Y=1)$, and $P(Xgeqslant2)=0$, the maximum is $$1-p+pe^{-p}$$
    $endgroup$
    – Did
    Jan 13 at 20:30












  • $begingroup$
    @Did That comment should be an answer, I think
    $endgroup$
    – leonbloy
    Jan 13 at 20:31










  • $begingroup$
    @Did Are we adding the lower probability for each case to ensure the equality? like.. adding the upper bound for each case?
    $endgroup$
    – JHK
    Jan 13 at 20:45










  • $begingroup$
    Use the remark that $$P(X=Y)=sum_nP(X=n,Y=n)leqslantsum_nmin{P(X=n),P(Y=n)}$$ hence in the present case, $$P(X=Y)leqslant P(X=0)+P(Y=1)$$ and to complete the proof, it remains to build $(X,Y)$ such that the inequality is an equality.
    $endgroup$
    – Did
    Jan 13 at 20:46


















$begingroup$
Since $P(X=0)=1-p<e^{-p}=P(Y=0)$, $P(X=1)=p>pe^{-p}=P(Y=1)$, and $P(Xgeqslant2)=0$, the maximum is $$1-p+pe^{-p}$$
$endgroup$
– Did
Jan 13 at 20:30






$begingroup$
Since $P(X=0)=1-p<e^{-p}=P(Y=0)$, $P(X=1)=p>pe^{-p}=P(Y=1)$, and $P(Xgeqslant2)=0$, the maximum is $$1-p+pe^{-p}$$
$endgroup$
– Did
Jan 13 at 20:30














$begingroup$
@Did That comment should be an answer, I think
$endgroup$
– leonbloy
Jan 13 at 20:31




$begingroup$
@Did That comment should be an answer, I think
$endgroup$
– leonbloy
Jan 13 at 20:31












$begingroup$
@Did Are we adding the lower probability for each case to ensure the equality? like.. adding the upper bound for each case?
$endgroup$
– JHK
Jan 13 at 20:45




$begingroup$
@Did Are we adding the lower probability for each case to ensure the equality? like.. adding the upper bound for each case?
$endgroup$
– JHK
Jan 13 at 20:45












$begingroup$
Use the remark that $$P(X=Y)=sum_nP(X=n,Y=n)leqslantsum_nmin{P(X=n),P(Y=n)}$$ hence in the present case, $$P(X=Y)leqslant P(X=0)+P(Y=1)$$ and to complete the proof, it remains to build $(X,Y)$ such that the inequality is an equality.
$endgroup$
– Did
Jan 13 at 20:46






$begingroup$
Use the remark that $$P(X=Y)=sum_nP(X=n,Y=n)leqslantsum_nmin{P(X=n),P(Y=n)}$$ hence in the present case, $$P(X=Y)leqslant P(X=0)+P(Y=1)$$ and to complete the proof, it remains to build $(X,Y)$ such that the inequality is an equality.
$endgroup$
– Did
Jan 13 at 20:46












0






active

oldest

votes











Your Answer





StackExchange.ifUsing("editor", function () {
return StackExchange.using("mathjaxEditing", function () {
StackExchange.MarkdownEditor.creationCallbacks.add(function (editor, postfix) {
StackExchange.mathjaxEditing.prepareWmdForMathJax(editor, postfix, [["$", "$"], ["\\(","\\)"]]);
});
});
}, "mathjax-editing");

StackExchange.ready(function() {
var channelOptions = {
tags: "".split(" "),
id: "69"
};
initTagRenderer("".split(" "), "".split(" "), channelOptions);

StackExchange.using("externalEditor", function() {
// Have to fire editor after snippets, if snippets enabled
if (StackExchange.settings.snippets.snippetsEnabled) {
StackExchange.using("snippets", function() {
createEditor();
});
}
else {
createEditor();
}
});

function createEditor() {
StackExchange.prepareEditor({
heartbeatType: 'answer',
autoActivateHeartbeat: false,
convertImagesToLinks: true,
noModals: true,
showLowRepImageUploadWarning: true,
reputationToPostImages: 10,
bindNavPrevention: true,
postfix: "",
imageUploader: {
brandingHtml: "Powered by u003ca class="icon-imgur-white" href="https://imgur.com/"u003eu003c/au003e",
contentPolicyHtml: "User contributions licensed under u003ca href="https://creativecommons.org/licenses/by-sa/3.0/"u003ecc by-sa 3.0 with attribution requiredu003c/au003e u003ca href="https://stackoverflow.com/legal/content-policy"u003e(content policy)u003c/au003e",
allowUrls: true
},
noCode: true, onDemand: true,
discardSelector: ".discard-answer"
,immediatelyShowMarkdownHelp:true
});


}
});














draft saved

draft discarded


















StackExchange.ready(
function () {
StackExchange.openid.initPostLogin('.new-post-login', 'https%3a%2f%2fmath.stackexchange.com%2fquestions%2f3072476%2fx-has-a-bernoulli-distribution-with-parameter-p-and-y-has-a-poisson-distributi%23new-answer', 'question_page');
}
);

Post as a guest















Required, but never shown

























0






active

oldest

votes








0






active

oldest

votes









active

oldest

votes






active

oldest

votes
















draft saved

draft discarded




















































Thanks for contributing an answer to Mathematics Stack Exchange!


  • Please be sure to answer the question. Provide details and share your research!

But avoid



  • Asking for help, clarification, or responding to other answers.

  • Making statements based on opinion; back them up with references or personal experience.


Use MathJax to format equations. MathJax reference.


To learn more, see our tips on writing great answers.




draft saved


draft discarded














StackExchange.ready(
function () {
StackExchange.openid.initPostLogin('.new-post-login', 'https%3a%2f%2fmath.stackexchange.com%2fquestions%2f3072476%2fx-has-a-bernoulli-distribution-with-parameter-p-and-y-has-a-poisson-distributi%23new-answer', 'question_page');
}
);

Post as a guest















Required, but never shown





















































Required, but never shown














Required, but never shown












Required, but never shown







Required, but never shown

































Required, but never shown














Required, but never shown












Required, but never shown







Required, but never shown







Popular posts from this blog

Mario Kart Wii

What does “Dominus providebit” mean?

Antonio Litta Visconti Arese